MGMAT CAT Esteria vs Burdistan

This topic has expert replies
Master | Next Rank: 500 Posts
Posts: 132
Joined: Thu Dec 02, 2010 2:49 am
Thanked: 5 times

MGMAT CAT Esteria vs Burdistan

by RadiumBall » Thu Mar 24, 2011 9:14 pm
The cost of manufacturing automobiles is considerably greater in Esteria than in Burdistan. In order to stimulate Esterian consumers' purchases of domestically manufactured automobiles, the Esterian government has historically charged taxes on automobiles manufactured in Burdistan. Five years ago, however, the Esterian government dropped those taxes; in those five years, the number of workers employed in Esterian automobile factories has decreased by 30%. Therefore, the number of vehicles manufactured and sold in Esteria must have decreased in the last five years.

Which of the following, if true, most weakens the argument?


A) Many Esterian automobile manufacturers operate factories outside Esteria.

B) The number of automobile workers in Burdistan has not increased during the same period.

C) Because vehicles manufactured in Esteria have a reputation for high quality, many Esterian consumers have continued to purchase domestically manufactured vehicles since the tax was abolished.

D) Esterian automobile manufacturers have lowered the price of their automobiles so as to remain competitive with the manufacturers in Burdistan.

E) Recent innovations in automated automobile-manufacturing technology have approximately halved the number of worker-hours required to produce most automobiles.

User avatar
Legendary Member
Posts: 1255
Joined: Fri Nov 07, 2008 2:08 pm
Location: St. Louis
Thanked: 312 times
Followed by:90 members

by Tani » Thu Mar 24, 2011 9:27 pm
The stimulus tells us that there are 30% fewer workers in the Esterian auto industry. From that the author concludes that there are fewer automobiles manufactured and sold in Esteria than before.

To weaken this, ask yourself what would be needed to show an equal or greater number of cars made and sold in Esteria despite fewer workers. The prediction: efficiency!

That's precisely what E gives us.

Look at the math. Say there were 100 auto worker 5 years ago, each working 2000 hours year (200,000 worker-hours total) and it took 200 worker-hours to produce a car. Then Esteria would have produced 1000 cars.

Now there are only 70 workers (140,000 worker-hours), but it only takes 100 worker hours to produce a car. Now Esteria can produce 1400 cars.
Tani Wolff

Master | Next Rank: 500 Posts
Posts: 132
Joined: Thu Dec 02, 2010 2:49 am
Thanked: 5 times

by RadiumBall » Thu Mar 24, 2011 9:43 pm
Perfectly right...but How did come to the conclusion that to weaken we need to show that an equal or greater number of cars made and sold in Esteria despite fewer workers?

1) How do you know that the car production is stable or increasing in Esteria?
2) Suppose we had a strengthen here what would be your problem statement? What we need to look for then?

Thanks

User avatar
Legendary Member
Posts: 1255
Joined: Fri Nov 07, 2008 2:08 pm
Location: St. Louis
Thanked: 312 times
Followed by:90 members

by Tani » Thu Mar 24, 2011 9:56 pm
The conclusion is that the number of cars made and sold in Esteria has decreased. We have to weaken the conclusion. To show that the number is not less we need to show that it is greater or equal. T


To strengthen we could turn the answer around and say efficiency is unchanged or even decreased.
Tani Wolff

Legendary Member
Posts: 1337
Joined: Sat Dec 27, 2008 6:29 pm
Thanked: 127 times
Followed by:10 members

by Night reader » Thu Mar 24, 2011 10:07 pm
past time premise
In order to stimulate Esterian consumers' purchases of domestically manufactured automobiles
minor conclusion-1, past time
the Esterian government has historically charged taxes on automobiles manufactured in Burdistan ...
premise-cause, past time
Five years ago, however, the Esterian government dropped those taxes

premise-effect, past time
Five years ago, ... in those five years, the number of workers employed in Esterian automobile factories has decreased by 30%.
current time premise
The cost of manufacturing automobiles is considerably greater in Esteria than in Burdistan.
Main Conclusion, past-present times continued
Therefore, the number of vehicles manufactured and sold in Esteria must have decreased in the last five years.
the conclusion contributing premise parts are selected in red

Which of the following, if true, most weakens the argument?


A) Many Esterian automobile manufacturers operate factories outside Esteria. - Irrelevant we are focusing on Esteria and not outside of Esteria

B) The number of automobile workers in Burdistan has not increased during the same period. - We still need to deal with the number of vehicles manufactured and sold in Esteria ...

C) Because vehicles manufactured in Esteria have a reputation for high quality, many Esterian consumers have continued to purchase domestically manufactured vehicles since the tax was abolished. - There's no indication of our contributing factors so far: abolishing of taxes, workers' number drop-down, cost of manufacturing

D) Esterian automobile manufacturers have lowered the price of their automobiles so as to remain competitive with the manufacturers in Burdistan. - There's no indication of competition in the argument, we have different factors contributing to the conclusion and we need to invalidate these factors

E) Recent innovations in automated automobile-manufacturing technology (cost related) have approximately halved the number of worker-hours required(workers' number related) to produce most automobiles. - Finally we can invalidate most of our contributing factors by using this answer choice.
My knowledge frontiers came to evolve the GMATPill's methods - the credited study means to boost the Verbal competence. I really like their videos, especially for RC, CR and SC. You do check their study methods at https://www.gmatpill.com

Newbie | Next Rank: 10 Posts
Posts: 1
Joined: Sun Dec 22, 2013 5:05 am

by JullsJulls » Fri Dec 27, 2013 12:58 am
The conclusion of this argument is : Therefore, the number of vehicles manufactured and sold in Esteria must have decreased in the last five years. Answer choice E addresses only the "manufacture vehicles but not sold. I understand that no response addresses these two issues simultaneously. However, why we should prefer E over other choices in this case?
Thanks

User avatar
GMAT Instructor
Posts: 3225
Joined: Tue Jan 08, 2008 2:40 pm
Location: Toronto
Thanked: 1710 times
Followed by:614 members
GMAT Score:800

by Stuart@KaplanGMAT » Fri Dec 27, 2013 7:20 pm
JullsJulls wrote:The conclusion of this argument is : Therefore, the number of vehicles manufactured and sold in Esteria must have decreased in the last five years. Answer choice E addresses only the "manufacture vehicles but not sold. I understand that no response addresses these two issues simultaneously. However, why we should prefer E over other choices in this case?
Thanks
Hi Julls!

This argument is a classic example of the most common pattern we see in critical reasoning: causation.

In a causal argument, the evidence describes some phenomenon and the conclusion is the author's explanation for that phenomenon. In every causal argument, the author makes 3 assumptions:

1) there's no other cause (most commonly tested);
2) it's not just a coincidence (fairly frequently tested); and
3) she doesn't have cause and effect backwards (rarely tested and nonsensical in many arguments).

The most common way to weaken a causal argument is to show that (1) is false: to identify another potential cause for what happened.

In this argument, what happened? In the last 5 years, the number of workers employed in E's auto factories has decreased by 30%. What's the author's explanation? The number of vehicles manufactured and sold must have decreased as well.

We predict: the correct answer will likely provide a different explanation for the decrease in workers.

(E) is a perfect match! If technological advances mean that we don't need as many workers for the same output, then now we know why the number of workers has decreased - and the number of cars produced could remain constant (or even have gone up, since workers produce double now and the number of workers has only decreased by 30%).

None of the other answers help explain the decrease in workers or attack the conclusion directly, so they're all out.

You specifically asked why does the correct answer only deal with manufacture, not sales? As long as we doubt one of those two, we doubt the conclusion (which is exactly what a weakener is supposed to do). Here's what the GMAT will NEVER do: give you two potential avenues of attack and have both appear in the choices, so you have to decide which one is more important. When there are multiple possible ways to weaken an argument (and there almost always are), only one will ever appear among the choices.

Hope that helps!

Stuart
Image

Stuart Kovinsky | Kaplan GMAT Faculty | Toronto

Kaplan Exclusive: The Official Test Day Experience | Ready to Take a Free Practice Test? | Kaplan/Beat the GMAT Member Discount
BTG100 for $100 off a full course

User avatar
Senior | Next Rank: 100 Posts
Posts: 90
Joined: Tue Aug 14, 2007 3:58 am
Thanked: 2 times

by kutlee » Thu May 14, 2015 9:14 am
In the option E, don't we have to make further assumption that Esterian automobile manufacturers have actually implemented the technological innovations and improved production?
Option C actually states that many Esterians continued buying their cars in spite of more imported cars available. The reason that number of workers has decreased by 30% may be due to excess inventory or some other reason.

User avatar
Legendary Member
Posts: 979
Joined: Tue Apr 14, 2009 1:38 am
Location: Hyderabad, India
Thanked: 49 times
Followed by:12 members
GMAT Score:700

by bubbliiiiiiii » Thu May 28, 2015 3:02 am
kutlee wrote: Option C actually states that many Esterians continued buying their cars in spite of more imported cars available. The reason that number of workers has decreased by 30% may be due to excess inventory or some other reason.
If that is the case inventory should have decreased as a result of heavy sales.

Hope it helps.
Regards,

Pranay

User avatar
Senior | Next Rank: 100 Posts
Posts: 90
Joined: Tue Aug 14, 2007 3:58 am
Thanked: 2 times

by kutlee » Thu May 28, 2015 7:04 am
Hi,
It is not necessary that heavy sales will lead to reduced inventory. What if production keeps up with demand?
Here option E wordings are not good.

Look at the following CR question
https://gmatclub.com/forum/from-1998-to- ... 59768.html
Here option C - Technological advancements in oil drilling techniques have allowed for a greater percentage of the world's oil to be obtained from underneath the ocean floor - is not correct. We cannot assume that Technological advancements are implemented by Country in question.

User avatar
Legendary Member
Posts: 979
Joined: Tue Apr 14, 2009 1:38 am
Location: Hyderabad, India
Thanked: 49 times
Followed by:12 members
GMAT Score:700

by bubbliiiiiiii » Thu May 28, 2015 7:31 am
kutlee wrote:Hi,
It is not necessary that heavy sales will lead to reduced inventory. What if production keeps up with demand?
Here option E wordings are not good.

Look at the following CR question
https://gmatclub.com/forum/from-1998-to- ... 59768.html
Here option C - Technological advancements in oil drilling techniques have allowed for a greater percentage of the world's oil to be obtained from underneath the ocean floor - is not correct. We cannot assume that Technological advancements are implemented by Country in question.
Hi Kutlee,

I think you are trying to draw an analogy between weaken and discrepancy question.
Regards,

Pranay

User avatar
Senior | Next Rank: 100 Posts
Posts: 90
Joined: Tue Aug 14, 2007 3:58 am
Thanked: 2 times

by kutlee » Thu May 28, 2015 9:51 pm
Logic is applied similar fashion to both. If you are to apply logic in both questions, you will have to extrapolate the information that some new technology is actually used. In the weakening argument, may be option E just raises a doubt. May be bit too weak.